0
$\begingroup$

I analyzed primes generated by applying four linear offsets to the first element of twin prime pairs ( 𝑝 , 𝑝 + 2 ), for 𝑝 ≀ 1,000,000:

π‘ž1 = 2𝑝 + 1, π‘ž2 = 2𝑝 + 7, π‘ž3 = 2𝑝 βˆ’ 3, π‘ž4 = 2𝑝 + 3.

About 57% of pairs produce at least one prime among these offsets. The rate varies by last digits of the twin primes: pairs ending in (1,3) and (7,9) have ~64% success, but those ending in (9,1) only ~43%.

I’d appreciate feedback on whether these patterns are known, and suggestions for related primes or offsets to investigate. Code and data are available here: https://github.com/allen-proxmire/twin-primes-offsets

$\endgroup$
2
  • 1
    $\begingroup$ It may be interesting to separate categories by position modulo a primorial, e.g., $6,30,210,2310,\dots$. Obviously there are effectively $100$% of twin prime pairs present in the single category modulo $6$, and I would expect that pairs are relatively evenly divided among the three categories modulo $30.$ $\endgroup$ Commented Aug 11 at 22:17
  • $\begingroup$ great idea. I haven't looked at mods yet. $\endgroup$ Commented Aug 12 at 15:30

1 Answer 1

1
$\begingroup$

In the (1,3) case we have:
$p \equiv 1 \mod 10 => q_4 \equiv 2 p + 3 \equiv 5 \mod 10$
So $q_4$ is never prime. While testing, I found that $q_1, q_2$ and $q_3$ produce primes.

In the (7,9) case we have:
$p \equiv 7 \mod 10 => q_1 \equiv 2 p + 1 \equiv 5 \mod 10$
So $q_1$ is never prime in this case. $q_2, q_3$ and $q_4$ produce primes.

In the (9,1) case we have:
$p \equiv 9 \mod 10 => q_2 \equiv 2 p + 7 \equiv 5 \mod 10$
$p \equiv 9 \mod 10 => q_3 \equiv 2 p - 3 \equiv 5 \mod 10$

So $q_2$ and $q_3$ are never prime in this case. $q_1$ and $q_4$ produce primes.

Since I found no restrictions for the other cases, in the long run I would suspect that (1,3) and (7,9) each produce 50% more primes among $q_1, q_2, q_3, q_4$ than (9,1).
This aligns pretty well with the 43% and 64%

$\endgroup$
1
  • $\begingroup$ I'm my GitHub you can find these results in a similar fashion. I breakdown how many primes are found by the four offsets. as Michael Stocker says, at least one offsets produces no primes in each of the last-digit-categories. initially I applied only two offsets to each group, but had a total of four different ones. applying all four, I found a few extra primes. thx $\endgroup$ Commented Aug 13 at 13:20

You must log in to answer this question.

Start asking to get answers

Find the answer to your question by asking.

Ask question

Explore related questions

See similar questions with these tags.